You are on page 1of 72

Question 1 of 55

A 61-year-old man presents with a two-week history of a sharp, stabbing pain over his right cheekbone. He describes the pain as
'very severe' and 'coming in spasms'. It typical lasts for around one minute before subsiding. The pain can be triggered by shaving
and eating. Examination of his eyes, cranial nerves and mouth is unremarkable. What is the most likely diagnosis?
Trigeminal neuralgia

Temporomandibular joint dysfunction

Temporal arteritis

Cluster headache

Herpes zoster ophthalmicus

Next question
The character of the pain in this patient is very typical of trigeminal neuralgia.

Trigeminal neuralgia
Trigeminal neuralgia is a pain syndrome characterised by severe unilateral pain. The vast majority of cases are idiopathic but
compression of the trigeminal roots by tumours or vascular problems may occur
The International Headache Society defines trigeminal neuralgia as:

a unilateral disorder characterised by brief electric shock-like pains, abrupt in onset and termination, limited to one or
more divisions of the trigeminal nerve
the pain is commonly evoked by light touch, including washing, shaving, smoking, talking, and brushing the teeth (trigger
factors), and frequently occurs spontaneously
small areas in the nasolabial fold or chin may be particularly susceptible to the precipitation of pain (trigger areas)
the pains usually remit for variable periods

Management

carbamazepine is first-line
failure to respond to treatment or atypical features (e.g. < 50 years old) should prompt referral to neurology

Trigeminal neuralgia - Summary


Trigeminal neuralgia occurs in the distribution of one or more branches of the fifth (trigeminal)
cranial nerve.
Pain may occur infrequently (e.g. periods of remission may last years) or with a frequency of up
to hundreds of times a day.
In 8090% of cases, trigeminal neuralgia is thought to be caused by compression of the
trigeminal nerve by a loop of artery or vein.

Almost twice as many women are affected by trigeminal neuralgia than men. The incidence
increases with age and is rare in people younger than 40 years of age.
Complications include depression, and inability to eat causing weight loss.
Features of trigeminal neuralgia include paroxysmal attacks of pain which may be precipitated
from trigger areas or by trigger factors.
There is no clinically evident neurological deficit.
Attacks are stereotyped in the individual patient.
'Atypical' or 'mixed' trigeminal neuralgia occurs when there is a persistent discomfort between
paroxysms or sensory loss.
The drug of choice for the treatment of trigeminal neuralgia is carbamazepine:
Dose should be titrated until pain is relieved. In the majority of people a dose of 200 mg three or
four times a day is sufficient to prevent paroxysms of pain (maximum dosage 1600 mg daily).
If carbamazepine is inappropriate, ineffective, or not tolerated, specialist advice should be sought,
or referral made to a neurologist or a specialist in pain management. A daily pain diary may be
useful to help people learn to manage their pain.
When the pain is in remission, the dose of carbamazepine should be reduced, and gradually
withdrawn if the person remains pain-free for 1 month.
Referral to a neurosurgeon, neurologist, or specialist in pain management (with an interest in
trigeminal neuralgia) should be made if any of the following apply:
They have severe pain.
Their pain significantly limits their daily activities and participation.
Atypical clinical features (e.g. burning pain between paroxysms, loss of sensation, abnormal
neurological signs) are present.
Carbamazepine is inappropriate, ineffective, or not tolerated.
Trigeminal neuralgia occurs in a person younger than 40 years of age.

Question 2 of 55
A 56-year-old woman presents with facial asymmetry. Whilst brushing her teeth this morning she noted that the right hand corner
of her mouth was drooping. She is generally well but noted some pain behind her right ear yesterday and says her right eye is
becoming dry. On examination she has a complete paralysis of the facial nerve on the right side, extending from the forehead to
the mouth. Ear, nose and throat examination is normal. Clinical examination of the peripheral nervous system is normal. What is
the most likely diagnosis?
Ramsey-Hunt syndrome

Bell's palsy

Stroke

Multiple sclerosis

Parotid tumour

Next question
The pain around the ear raises the possibility of Ramsey-Hunt syndrome but this is actually quite common in Bell's palsy - some
studies suggest it is seen in 50% of patients. The normal ear exam also goes against this diagnosis.

Bell's palsy
Bell's palsy may be defined as an acute, unilateral, idiopathic, facial nerve paralysis. The aetiology is unknown although the role of
the herpes simplex virus has been investigated previously.
Features

lower motor neuron facial nerve palsy - forehead affected*


patients may also notice post-auricular pain (may precede paralysis), altered taste, dry eyes, hyperacusis

Management

in the past a variety of treatment options have been proposed including no treatment, prednisolone only and a
combination of aciclovir and prednisolone
following a National Institute for Health randomised controlled trial it is now recommended that prednisolone 25mg bd for
10 days should be prescribed for patients within 72 hours of onset of Bell's palsy. Adding in aciclovir gives no additional
benefit
eye care is important - prescription of artificial tears and eye lubricants should be considered

Prognosis

if untreated around 15% of patients have permanent moderate to severe weakness

*upper motor neuron lesion 'spares' upper face

Bell's palsy - Summary


Bell's palsy is an acute, unilateral, idiopathic, facial nerve paralysis. The paralysis also affects the
eyelids, causing an impairment of blinking.

The cause of Bell's palsy remains unclear, but it has been associated with the herpes virus.
The annual incidence of Bell's palsy in the UK is 1 in 5000 people. It occurs most commonly
between 15 and 60 years of age.
When making a diagnosis of Bells palsy:
It should be noted that maximum facial weakness develops within 2 days. Earache, pain behind
the ear, aural fullness, or facial pain may precede the palsy. Severe pain might indicate Ramsay
Hunt syndrome. This is caused by herpes zoster and is associated with a painful rash and herpetic
vesicles.
Confirmation that the paralysis is caused by a unilateral, lower motor neuron lesion is required.
The muscles controlling facial expression are affected on one side of the face only. This may
result in drooping of the brow and corner of the mouth, weakness of the frontalis (forehead
muscle), or inability to close the eye.
It should be confirmed that only the facial nerve is affected.
Other features such as loss of taste of the anterior two-thirds of the tongue (on the same side as
the facial weakness) may occur.
Serious underlying pathology such as Lyme disease and parotid tumour should be excluded.
To manage Bells palsy:
The person should be reassured that the prognosis is good: most people with Bell's palsy make a
full recovery within 9 months.
The person should be advised to keep the affected eye lubricated by using lubricating eye drops
during the day and ointment at night. The eye should be taped closed at bedtime using
microporous tape, if the ability to close the eye at night is impaired.
For people presenting within 72 hours of the onset of symptoms, prescription of prednisolone
should be considered. There are no data supporting the use of prednisolone in people presenting
after 72 hours.
Antiviral treatment is not recommended, either alone or in combination with prednisolone.
If there is doubt regarding the diagnosis, or there is recurrent or bilateral Bell's palsy, an urgent
referral to neurology or to an ear, nose, and throat (ENT) specialist should be arranged.
If the cornea remains exposed after attempting to close the eyelid, an urgent referral to
ophthalmology should be arranged.

If the paralysis shows no sign of improvement after one month, or there is suspicion of a serious
underlying diagnosis (e.g. cholesteatoma, parotid tumour, malignant otitis externa), an urgent
referral to ENT should be arranged.
If there is residual paralysis after 69 months, referral to a plastic surgeon with a special interest
in facial reconstructive surgery should be considered.
Question 3 of 55
You review a 23-year-old woman who presents with a three week history of bilateral nasal obstruction, cough at night and a clear
nasal discharge. She had similar symptoms around this time last year and the only history of note is asthma. What is the most
likely diagnosis?
Allergic rhinitis

Chronic sinusitis

Nasal hypertrophy secondary to the steroid inhaler

Nasal polyps

Vasomotor rhinitis

Next question

Allergic rhinitis
Allergic rhinitis is an inflammatory disorder of the nose where the nose become sensitized to allergens such as house dust mites
and grass, tree and weed pollens. It may be classified as follows, although the clinical usefulness of such classifications r emains
doubtful:

seasonal: symptoms occur around the same time every year. Seasona l rhinitis which occurs secondary to pollens is
known as hay fever
perennial: symptoms occur throughout the year
occupational: symptoms follow exposure to particular allergens within the work place

Features

sneezing
bilateral nasal obstruction
clear nasal discharge
post-nasal drip
nasal pruritus

Management of allergic rhinitis

allergen avoidance
oral or intranasal antihistamines are first line
intranasal corticosteroids

course of oral corticosteroids are occasionally needed


there may be a role for short courses of topical nasal decongestants (e.g. oxymetazoline). They should not be used for
prolonged periods as increasing doses are required to achieve the same effect (tachyphylaxis) and rebound hypertrophy
of the nasal mucosa may occur upon withdrawal

Allergic rhinitis - Summary


Allergic rhinitis is an inflammatory disorder of the nose which occurs when the membranes lining
the nose become sensitized to allergens.
People may be allergic to one or more allergens, which complicates their identification by history
alone. The most common allergens include:
House dust mites.
Grass, tree, and weed pollens.
Allergens carried on animal hair.
Allergens encountered at work such as latex gloves and wood dust.
The symptoms of many people with seasonal allergic rhinitis improve over time. Some people
develop asthma, eczema, or allergic conjunctivitis.
A person with rhinitis typically presents with sneezing, itching, and nasal discharge and/or
blockage.
Allergy
testing should be arranged when the type of rhinitis (infective, irritant, or allergic) is unclear from
the history, or if symptoms are persistent or poorly controlled.
Referral for skin prick testing should be arranged, if available. Results can be suppressed by some
drugs (e.g.antihistamines and topical corticosteroids).
If skin prick testing is not available or suitable, blood should be sent for serum total and specific
immunoglobulin (Ig) E testing. If allergy testing is negative, non-allergic causes for the rhinitis
(such as drugs and hormonal causes) should be considered.
Allergen avoidance is fundamental to the management of allergic rhinitis; however, drug
treatment may be necessary. If nasal drops or a spray is prescribed, an explanation should be
given of the importance of good technique.
For 'as-required' treatment for occasional symptoms, an antihistamine should be prescribed.

For people with allergic conjunctivitis, children aged 2 to 5 years of age, and people who prefer
oral treatment, an oral antihistamine should be prescribed first line.
For all other people, intranasal azelastine should be prescribed first line.
For people who want preventive treatment to control more frequent or persistent symptoms, the
importance of regular treatment should be explained.
If the predominant symptom is nasal blockage, or nasal polyps are present, an intranasal
corticosteroid should be prescribed.
If the predominant symptoms are sneezing or nasal discharge, either an oral antihistamine or an
intranasal corticosteroid should be prescribed.
For pregnant or breastfeeding women, an intranasal corticosteroid should be prescribed first line.
For people who require rapid relief of symptoms while awaiting preventive treatment to take
effect:
If nasal congestion is a problem, an intranasal decongestant should be prescribed.
If using an intranasal corticosteroid, an oral antihistamine should be prescribed.
If symptoms are severe and impairing quality of life, a course of oral prednisolone should be
prescribed.
People should be advised to reconsult after 24 weeks if symptoms remain inadequately
controlled.
Treatment should be continued until they are no longer likely to be exposed to the suspected
allergen.
People with recurrent episodes of allergic rhinitis controlled by intranasal corticosteroids should
be advised to restart treatment at least 7 days before re-exposure to allergen. When the time of re
exposure to the antigen is uncertain, treatment should be started several weeks before the most
likely time of reexposure.
If there is inadequate benefit with current treatment, step-up treatment should be considered.

Question 1 of 52
A 23-year-old man is diagnosed as having nasal polyps. Sensitivity to which medication is associated with this condition?

Sulfa drugs

ACE inhibitors

Penicillins

Paracetamol

Aspirin

Next question

Nasal polyps
Around in 1% of adults in the UK have nasal polyps. They are around 2 -4 times more common in men and are not commonly seen
in children or the elderly.
Associations

asthma* (particularly late-onset asthma)


aspirin sensitivity*
infective sinusitis
cystic fibrosis
Kartagener's syndrome
Churg-Strauss syndrome

Features

nasal obstruction
rhinorrhoea, sneezing
poor sense of taste and smell

Unusual features which always require further investigation include unilateral symptoms or bleeding.
Management

all patients with suspected nasal polyps should be referred to ENT for a full examination
topical corticosteroids shrink polyp size in around 80% of patients

*the association of asthma, aspirin sensitivity and nasal polyposis is known as Samter's triad

Question 1 of 51
A 27-year-old woman complains of recurrent ear discharge. Otoscopy is as follows:

What is the most likely diagnosis?


Otitis externa

Chronic suppurative otitis media

Mastoiditis

Cholesteatoma

Acute otitis media

Next question

Cholesteatoma
A cholesteatoma consists of squamous epithelium that is 'trapped' within the skull base
Main features

foul smelling discharge


hearing loss

Other features are determined by local invasion:

vertigo
facial nerve palsy
cerebellopontine angle syndrome

Otoscopy

'attic crust' - seen in the uppermost part of the ear drum

Question 2 of 51
A 57-year-old woman presents with an 8 week history of intermittent dizziness. These episodes typically occur when she sud denly
moves her head and are characterised by the sensation that the room is 'spinning'. Most attacks last around one minute before
dissipating. Neurological examination is unremarkable. What is the most likely diagnosis?
Benign paroxysmal positional vertigo

Meniere disease

Crescendo transient ischaemic attacks

Multiple sclerosis

Viral labyrinthitis

Next question

Viral labyrinthitis typically causes constant symptoms of a shorter duration. Patients with Meniere disease usually have associated
hearing loss and tinnitus. Also, the vertigo associated with Meniere disease typically lasts much longer.

Benign paroxysmal positional vertigo


Benign paroxysmal positional vertigo (BPPV) is one of the most common causes of vertigo encountered. It is characterised by the
sudden onset of dizziness and vertigo triggered by changes in head position. The average age of onset is 55 years and it is l ess
common in younger patients.
Features

vertigo triggered by change in head posi tion (e.g. rolling over in bed or gazing upwards)
may be associated with nausea
each episode typically lasts 10-20 seconds
positive Dix-Hallpike manoeuvre

BPPV has a good prognosis and usually resolves spontaneously after a few weeks to months. Symptomati c relief may be gained
by:

Epley manoeuvre (successful in around 80% of cases)


teaching the patient exercises they can do themselves at home, for example Brandt -Daroff exercises

Medication is often prescribed (e.g. Betahistine) but it tends to be of limited value.

Benign paroxysmal positional vertigo - Summary


Benign paroxysmal positional vertigo (BPPV) is defined as 'a disorder of the inner ear
characterized by repeated episodes of positional vertigo' (symptoms occur with changes in the
position of the head).
It is thought to be caused by otolith particles becoming dislodged from the otolithic membranes
and falling into the semicircular canals (usually the posterior canal) and causing movement of the
fluid of the inner ear.
It commonly occurs spontaneously, particularly in older people, but can be precipitated by a head
injury, ear surgery, prolonged bed rest, or following an episode of any inner ear pathology, e.g.
labyrinthitis. It may coexist with other inner ear conditions, e.g. Menieres disease.
BPPV can affect people of any age, but commonly presents at around 50 years of age. Women
are affected twice as often as men.
BPPV often has a relapsing and remitting course with long asymptomatic periods. Recovery can
occur spontaneously without treatment, although recurrence is common.
To diagnose BPPV:
Symptoms of vertigo should be confirmed. Symptoms are brought on by specific movements of
the head (e.g. turning over in bed, looking upwards, or bending over).
Other causes of vertigo should be considered including Menieres disease, postural hypotension,
and anxiety disorder.
Imaging (e.g. magnetic resonance imaging or computed tomography) is not required to confirm
the diagnosis of BPPV unless it is necessary to exclude another condition (e.g. if the person has
atypical nystagmus or additional neurological symptoms).
To manage BPPV:
The option of watchful waiting should be discussed to see whether symptoms settle without
treatment. An explanation that treatment may help the person's symptoms resolve more quickly
should be offered.
If the person prefers treatment, a particle repositioning manoeuvre, such as the Epley manoeuvre
should be offered and Brandt-Daroff exercises considered. Symptomatic drug treatment is not
usually helpful for people with BPPV.

The person should be advised to return for follow up in 4 weeks if symptoms have not resolved in
case BPPV has been incorrectly diagnosed.
Admission to hospital should be arranged if there is severe nausea and vomiting and an inability
to tolerate oral fluids.
Referral should be arranged to a medically qualified balance specialist (such as an ear, nose, and
throat specialist, audiovestibular specialist physician, or care of the elderly physician with a
special interest depending on local protocol) if any of the following apply:
The expertise to provide the Epley manoeuvre is not available in primary care.
The Epley manoeuvre has been performed and repeated, and symptoms are still present.
Symptoms or signs are atypical.
Symptoms and signs have not resolved in 4 weeks.
There have been three or more periods during which the person has experienced episodes of
vertigo.
Question 3 of 51
A 30-year-old woman is admitted to the Emergency Department following a suspected p eanut allergy. On examination she has
gross facial and tongue oedema. Her oxygen saturations are 97% on room air, pulse is 96 / min and blood pressure is 90/62
mmHg. The paramedics have already gained intravenous access. What is the most appropriate way to give adrenaline in this
situation?
Nebulised

Subdermally

Intramuscularly

Intravenously

Subcutaenously

Next question
The Resuscitation Council guidelines only recommend giving adrenaline intramuscularly, regardless of whether the patient has
intravenous access or not.

Anaphylaxis
Anaphylaxis may be defined as a severe, life-threatening, generalised or systemic
hypersensitivity reaction.
Anaphylaxis is one of the few times when you would not have time to look up the dose of a medication. The Resuscitation Counc il

guidelines on anaphylaxis have recently been updated. Adrenaline is by far the most important drug in anaphylaxis and should be
given as soon as possible. The recommended doses for adrenaline, hydrocortisone and chlorphenamine are as follows:
Adrenaline
Hydrocortisone Chlorphenamine
150 mcg (0.15ml 1 in 1,000)25 mg
250 mcg/kg
< 6 months
150 mcg (0.15ml 1 in 1,000)50 mg
2.5 mg
6 months - 6 years
300 mcg (0.3ml 1 in 1,000) 100 mg
5 mg
6-12 years
10 mg
Adult and child > 12 years 500 mcg (0.5ml 1 in 1,000) 200 mg
Adrenaline can be repeated every 5 minutes if necessary. The best site for IM injection is the anterolateral aspect of the middle
third of the thigh.
Common identified causes of anaphylaxis

food (e.g. Nuts) - the most common cause in children


drugs
venom (e.g. Wasp sting)

Question 4 of 51
A 23-year-old woman presents one week after being prescribed a combined antibiotic and steroid spray for otitis externa. There
has been no improvement in her symptoms and the erythema seems to have extended to the ear itself. What is the most
appropriate treatment?
Topical clotrimazole

Oral flucloxacillin

Topical ciprofloxacin

Oral fluconazole

Oral ciprofloxacin

Next question
The spreading erythema is an indication for oral antibiotics. Flucloxacillin is first -line.

Otitis externa
Otitis externa is a common reason for primary care attendance in the UK.
Causes of otitis externa include:

infection: bacterial (Staphylococcus aureus, Pseudomonas aeruginosa) or fungal


seborrhoeic dermatitis
contact dermatitis (allergic and irritant)

Features

ear pain, itch, discharge


otoscopy: red, swollen, or eczematous canal

The recommend initial management of otitis externa is:

topical antibiotic or a combined topical antibiotic with steroid


if the tympanic membrane is perforated aminoglycosides are traditionally not used*
if there is canal debris then consider removal
if the canal is extensively swollen then an ear wick is sometimes inserted

Second line options include

consider contact dermatitis secondary to neomycin


oral antibiotics if the infection is spreading
taking a swab inside the ear canal
empirical use of an antifungal agent

Malignant otitis externa is more common in elderly diabetics. In this condition there is extension of infection into the bony ear canal
and the soft tissues deep to the bony canal. Intravenous antibiotics may be required.
*many ENT doctors disagree with this and feel that concerns about ototoxicity are unfounded

Otitis externa - Summary


Otitis externa is inflammation of the external ear canal and can be classed as acute (lasting < 3
weeks) or chronic (lasting > 3 months).
Localized otitis externa is an infection of a hair follicle that can progress to become a boil in the
ear canal.
Diffuse otitis externa is more widespread inflammation of the skin and subdermis of the external
ear canal.
Malignant otitis externa is an extension of otitis externa into the bone surrounding the ear canal
(i.e. the mastoid and temporal bones). Malignant otitis, without treatment, is a fatal condition.
Osteomyelitis will progressively involve the mastoid, temporal, and basal skull bones, and the
infection will spread to cerebrospinal fluid causing meningitis.
Otitis externa is slightly more common in women than in men, the prevalence peaking in women
aged 4554 years and in men aged 6574 years.
Direct causes of otitis externa include infection and dermatitis.
Precipitating factors for otitis externa include ear trauma and excessive moisture. Risk factors for
malignant otitis include diabetes and radiotherapy to head and neck.
Complications include abscess, inflammation of the tympanic membrane, and malignant otitis.

Diagnosis of otitis externa is based on characteristic symptoms and signs. Alternative diagnoses
(such as otitis media, earwax, and neoplasm) can usually be differentiated by the history (e.g.
rapidity of onset, duration, pattern of symptoms) and clinical examination.
Management of otitis externa includes:
Assessing the severity of symptoms (e.g. pain, itch, hearing loss, and ear discharge).
Removing or treating any aggravating or precipitating factors (such as diabetes, dermatitis, and
trauma to the ear).
Offering paracetamol or ibuprofen for symptomatic relief (with codeine for severe pain).
Treating infection, if necessary, usually with a topical preparation.
Considering the need for investigations. Investigations are rarely useful but may be necessary if
symptoms are persistent or recurrent.
Providing appropriate self-care advice to aid recovery and reduce risk of recurrence (e.g. keeping
the ears dry and clean, avoiding the use of cotton buds, treating generalized skin conditions such
as eczema).
For localized otitis externa:
Application of local heat (e.g. from a warm flannel) may be sufficient as folliculitis is usually
mild and self limiting.
If pus is causing severe pain and swelling, incision and drainage should be considered. Incision
and drainage usually requires referral, although a small pustule near the entrance to the ear canal
may be drained by incising it with a surgical needle.
If malignant otitis is suspected, urgent admission should be arranged.
Specialist advice should be considered if symptoms are persistent, contact sensitivity is suspected,
or the ear canal is occluded.
Referral to secondary care should be considered if there is extensive cellulitis, extreme pain or
discomfort, considerable discharge or extensive swelling of the auditory canal, or sufficient
earwax or debris to obstruct topical medication.
Follow up is not usually needed; however, it is recommended:
After completion of a course of treatment, to review response.

In people with diabetes or compromised immunity.


In people with severe otitis externa with accompanying cellulitis which has spread outside the
auditory canal.

Question 5 of 51
A 35-year-old man presents to his GP surgery as he is having some difficulties with his hearing. He now struggles to follow
conversation and often has the TV volume turned up high. Otoscopy is normal. An audiogram is requested:

What does the audiogram show?

Bilateral mixed hearing loss

Right conductive hearing loss

Normal hearing

Bilateral conductive hearing loss

Bilateral sensorineural hearing loss

Next question

Audiograms
Audiograms are usually the first-line investigation that is performed when a patient complains of hearing difficulties. They are
relatively easy to interpret as long as some simple rules are followed:

anything above the 20dB line is essentially normal (marked in red on the blank audiogram below)
in sensorineural hearing loss both air and bone conduction are impaired
in conductive hearing loss only air conduction is impaired
in mixed hearing loss both air and bone conduction are impaired, with air conduction often being 'worse' than bone

Question 6 of 51
A 41-year-old complains of some difficulties with his hearing which have got gradually worse over the past few weeks. An
audiogram is arranged:

What does the audiogram show?


Right sensorineural hearing loss

Right conductive hearing loss

Bilateral mixed hearing loss

Right mixed hearing loss

Spurious results - need to repeat

Next question

Question 7 of 51
A 3-year-old boy is brought to surgery. His mum reports that he has been complaining of a sore left ear for the past 2 -3 weeks.
This morning she noticed some 'green gunge' on his pillow. On examination his temperature is 37.8C. Otoscopy of the right ea r is
normal. On the left side the tympanic membrane cannot be visualised as the ear canal is full with a yellow-green discharge. What
is the most appropriate action?
Review in 2 weeks

Admit to paediatrics

Advise olive oil drops followed by ear syringing

Urgent referral to ENT

Amoxicillin + review in 2 weeks

Next question

This boy is likely to have had an acute otitis media with perforation.

Perforated tympanic membrane


The most common cause of a perforated tympanic membrane is infection. Other causes include barotrauma or direct trauma.
A perforated tympanic membrane may lead to hearing loss depending on the size and also increase the risk of otitis media.
Management

no treatment is needed in the majority of cases as the tympanic membrane will usually heal after 6 -8 weeks. It is
advisable to avoid getting water in the ear during this time
it is common practice to prescribe antibiotics to perforations which occur following an episode of acute otitis media. NICE
support this approach in the 2008 Respiratory tract infection guidelines
myringoplasty may be performed if the tympanic membrane does not heal by itself

Question 8 of 51
A 22-year-old man complains of hearing problems. You perform an examination of his auditory system including Rinne's and
Weber's test:

Rinne's test :

Weber's test:

L eft ear: bone conduction > air c onduction


Right ear: air c onduction > bone conduction
L ateralises to the left s ide

What do these tests imply?


Normal hearing

Left conductive deafness

Right conductive deafness

Left sensorineural deafness

Right sensorineural deafness

Next question

Rinne's and Weber's test


Performing both Rinne's and Weber's test allows differentiation of conductive and sensorineural deafness.
Rinne's test

tuning fork is placed over the mastoid process until the sound is no longer heard, followed by repositioning just over
external acoustic meatus
air conduction (AC) is normally better than bone conduction (BC)
if BC > AC then conductive deafness

Weber's test

tuning fork is placed in the middle of the forehead equidistant from the patient's ears
the patient is then asked which side is loudest
in unilateral sensorineural deafness, sound is localised to the unaffected side
in unilateral conductive deafness, sound is localised to the affected side

Question 9 of 51
A 6-year-old boy is brought to surgery. His mother says he has been complaining of left sided otalgia for the past three days.
Otoscopy demonstrates the following:

What is the most likely diagnosis?


Acute otitis media with perforation

Cholesteatoma

Glue ear

Normal tympanic membrane

Acute otitis media

Next question
The bulging nature of the tympanic membrane strongly suggests a diagnosis of otitis media. The colour of the tympanic
membrane alone has a low predictive value for otitis media as it may be reddened by coughing, nose blowing, and fever.

Otitis media
Following the 2008 NICE guidelines on respiratory tract infections antibiotics are not routinely recommended. NICE recommends
however that they should be considered in the following situations:

children younger than 2 years with bilateral acute otitis media


children with otorrhoea who have acute otitis media

Question 10 of 51
A 14-year-old male presents to surgery with a 3 day history of a sore throat. Which one of the following features is not an
indication for antibiotic therapy?
Temperature of 39.1C

A past history of diabetes mellitus

Two previous episodes in the past 5 months

Unilateral peritonsillitis on examination

A past history of rheumatic fever

Next question

A temperature of 39.1C would indicate marked systemic upset

Sore throat
Sore throat encompasses pharyngitis, tonsillitis, laryngitis
Clinical Knowledge Summaries recommend:

throat swabs and rapid antigen tests should not be carried out routinely in patients with a sore throat

Management

paracetamol or ibuprofen for pain relief


antibiotics are not routinely indicated

NICE indications for antibiotics

features of marked systemic upset secondary to the acute sore throat


unilateral peritonsillitis
a history of rheumatic fever
an increased risk from acute infection (such as a child with diabetes mellitus or immunodeficiency)

patients with acute sore throat/acute pharyngitis/acute tonsillitis when 3 or more Centor criteria are present

The Centor criteria* are as follows:

presence of tonsillar exudate


tender anterior cervical lymphadenopathy or lymphadenitis
history of fever
absence of cough

If antibiotics are indicated then either phenoxymethylpenicillin or erythromycin (if the patient is penicillin allergic) shou ld be given.
Either a 7 or 10 day course should be given
*if 3 or more of the criteria are present there is a 40 -60% chance the sore throat is caused by Group A beta -haemolytic
Streptococcus

Question 1 of 41
A 59-year-old man presents with a severe pain deep within his right ear. He feels dizzy and reports that the room 'is spinning'.
Clinical examination shows a partial facial nerve palsy on the right side and vesicular lesions on the anterior two -thirds of his
tongue. What is the most likely diagnosis?
Meniere's disease

Herpes zoster ophthalmicus

Ramsay Hunt syndrome

Acoustic neuroma

Trigeminal neuralgia

Next question

Whilst vesicular lesions are more classically seen in the external auditory canal and pinna they may also be seen on the anterior
2/3rds of the tongue and the soft palate.

Ramsay Hunt syndrome


Ramsay Hunt syndrome (herpes zoster oticus) is caused by the reactivation of the varicella zoster virus in the geniculate gan glion
of the seventh cranial nerve.
Features

auricular pain is often the first feature


facial nerve palsy
vesicular rash around the ear
other features include vertigo and tinnitus

Management

oral aciclovir and corticosteroids are usually given

Question 2 of 41
A 40-year-old woman presents with recurrent episodes of vertigo associated with a feeling or 'fullness' and 'pressure' in her ears.
She thinks her hearing is worse during these attacks. Clinical examination is unremarkable. What is the most likely diagnosis?
Meniere's disease

Benign paroxysmal positional vertigo

Acoustic neuroma

Cholesteatoma

Somatisation

Next question

Meniere's disease
Meniere's disease is a disorder of the inner ear of unknown cause. It is characterised by excessive pressure and progressive
dilation of the endolymphatic system. It is more common in middle -aged adults but may be seen at any age. Meniere's disease
has a similar prevalence in both men and women.
Features

recurrent episodes of vertigo, tinnitus and hearing loss (sensorineural). Vertigo is usually the prominent symptom
a sensation of aural fullness or pressure is now recognised as being common
other features include nystagmus and a positive Romberg test
episodes last minutes to hours
typically symptoms are unilateral but bilateral symptoms may develop after a number of years

Natural history

symptoms resolve in the majority of patients after 5 -10 years


some patients may be left with hearing loss
psychological distress is common

Management

ENT assessment is required to confirm the diagnosis


patients should inform the DVLA. The current advice is to cease driving until satisfactory control of symptoms is achieved
acute attacks: buccal or intramuscular prochlorperazine. Admission is sometimes requ ired
prevention: betahistine may be of benefit

Meniere's disease - Summary


Meniere's disease is a rare progressive disorder of the inner ear of unknown cause characterized
by recurrent acute episodes of vertigo, hearing loss, tinnitus, and a sense of pressure in the ear
(aural fullness). Vertigo (causing dizziness, nausea, and vomiting) is often the most prominent
symptom.

The cause of Meniere's disease is not known but is likely to be multifactorial and associated with
endolymphatic hydrops (raised endolymph pressure in the membranous labyrinth of the inner
ear).
Suggested risk factors include:
Allergy (for example food allergy).
Autoimmunity.
Genetic susceptibility.
Metabolic disturbances involving the balance of sodium and potassium in the fluid of the inner
ear.
Vascular factors (there is an association between migraine and Meniere's disease).
Viral infection.
The true incidence of Meniere's disease is not known because the diagnostic criteria have been
used inconsistently, although it is estimated that the prevalence in the UK is about 1 in 1000
people and women are possibly affected slightly more often than men.
There are few major complications of Menieres disease. Some people will develop progressive
hearing loss (which may be bilateral), but resolution of vertigo occurs in around three-quarters of
people over a 5-10 year time period.
A firm diagnosis of Meniere's disease requires all three of the following criteria:
Vertigo at least two spontaneous episodes lasting at least 20 minutes within a single attack of
Meniere's disease.
Tinnitus and/or perception of aural fullness.
Hearing loss confirmed by audiometry to be sensorineural in nature.
Confirmation of the diagnosis requires referral to an Ear, Nose, and Throat (ENT) consultant and
a formal audiology assessment.
People who have frequent, sudden attacks should be advised to keep medication readily
accessible, and to consider the risks before starting potentially dangerous activities like driving,
swimming, or operating machinery.

To help alleviate nausea, vomiting, and vertigo in people with acute Meniere's disease, a 7 day
(14 days if required previously) course of prochlorperazine or an antihistamine (e.g. cinnarizine,
cyclizine, or promethazine teoclate) should be considered.
If symptoms are severe enough, people may require hospital admission for intravenous (IV)
labyrinthine sedatives and fluids to maintain hydration and nutrition.
A trial of betahistine can be considered to reduce the frequency and severity of attacks of hearing
loss, tinnitus, and vertigo.

Question 3 of 41
Which one of the following is least recognised as a cause of vertigo?
Gentamicin

Meniere's disease

Acoustic neuroma

Multiple sclerosis

Motor neuron disease

Next question

Vertigo
The table below lists the main characteristics of the most important causes of vertigo
Disorder
Viral labyrinthitis

Vestibular neuritis
Benign paroxysmal positional
v ertigo
Meniere's disease
Vertebrobasilar ischaemia
Acoustic neuroma

Other causes of vertigo incl ude

Notes
Recent viral infection
Sudden onset
Nausea and vomiting
Hearing may be affected
Recent viral infection
Recurrent vertigo attacks lasting hours or days
No hearing loss
Gradual onset
Triggered by change in head position
Each episode lasts 10-20 seconds
Associated with hearing loss, tinnitus and sensation of fullness or pressure in one or both
ears
Elderly patient
Dizziness on extension of neck
Hearing loss, vertigo, tinnitus
Absent corneal reflex is important sign
Associated with neurofibromatosis type 2

trauma
multiple sclerosis
ototoxicity e.g. gentamicin

Question 4 of 41
A 37-year-old man presents with nasal obstruction and loud snoring. He has noticed these symptoms get gradually worse for the
past two months. His left nostril feels blocked whilst his right feels clear and normal. There is no history of epistaxis and he is
systemically well. On examination a large nasal polyp can be seen in the left nostril. What is the most appropriate action?
Reassure + provide patient information leaflet on nasal polyps

Enquire about cocaine use

Refer to ENT

Trial of intranasal steroids

Nasal cautery

Next question
Given that his symptoms are unilateral it is important he is referred to ENT for a full examination.

Nasal polyps
Around in 1% of adults in the UK have nasal polyps. They are around 2 -4 times more common in men and are not commonly seen
in children or the elderly.
Associations

asthma* (particularly late-onset asthma)


aspirin sensitivity*
infective sinusitis
cystic fibrosis
Kartagener's syndrome
Churg-Strauss syndrome

Features

nasal obstruction
rhinorrhoea, sneezing
poor sense of taste and smell

Unusual features which always require further investigation include unilateral symptoms or bleeding.
Management

all patients with suspected nasal polyps should be referred to ENT for a full examination
topical corticosteroids shrink polyp size in around 80% of patients

*the association of asthma, aspirin sensitivity and nasal polyposis is known as Samter's triad

Question 5 of 41
A patient presents due to a 'brown coating' on his tongue. He is 34-years-old and has no significant medical history. The coating
has been present for the past few weeks. He is asymptomatic other than a slight 'tickling' sensation on his tongue.

What is the most likely diagnosis?


Lichen Planus

Oral Candida

Iron-deficiency anaemia

Hairy leukoplakia

Black hairy tongue

Next question

Black hairy tongue


Black hairy tongue is relatively common condition which results from defective desquamation of the filiform papillae. Despite the
name the tongue may be brown, green, pink or another colour.
Predisposing factors

poor oral hygiene


antibiotics
head and neck radiation
HIV
intravenous drug use

The tongue should be swabbed to exclude Candida


Management

tongue scraping
topical antifungals if Candida

Question 6 of 41
A 25-year-old woman presents as she has noticed an unusual appearance of her tongue. This has been present for the past few
weeks. She reports getting a burning sensation when she eats spicy food.

What is the most likely diagnosis?


Strawberry tongue

Geographic tongue

Hairy leukoplakia

Oral Candida

Glossitis likely secondary to anaemia

Next question

Geographic tongue
Geographic tongue is a benign, chronic condition of unknown cause. It is present in around 1-3% of the population and is more
common in females.
Features

erythematous areas with a white-grey border (the irregular, smooth red areas are said to look like the outline of a map)
some patients report burning after eating certain food

Management

reassurance about benign nature

Question 7 of 41
Which one of the following is least recognised in patients with Meniere's disease
Aural fullness

Symptoms triggered by sudden change in head position

Sensorineural hearing loss

Tinnitus

Nystagmus

Next question

Meniere's disease
Meniere's disease is a disorder of the inner ear of unknown cause. It is characterised by excessive pressure and progressive
dilation of the endolymphatic system. It is more common in middle-aged adults but may be seen at any age. Meniere's disease
has a similar prevalence in both men and women.
Features

recurrent episodes of vertigo, tinnitus and hearing loss (sensorineural). Vertigo is usually the promin ent symptom
a sensation of aural fullness or pressure is now recognised as being common
other features include nystagmus and a positive Romberg test
episodes last minutes to hours
typically symptoms are unilateral but bilateral symptoms may develop after a number of years

Natural history

symptoms resolve in the majority of patients after 5 -10 years


some patients may be left with hearing loss
psychological distress is common

Management

ENT assessment is required to confirm the diagnosis


patients should inform the DVLA. The current advice is to cease driving until satisfactory control of symptoms is achieved
acute attacks: buccal or intramuscular prochlorperazine. Admission is sometimes required
prevention: betahistine may be of benefit

Question 8 of 41

A 52-year-old woman presents to surgery with a two week history of dizziness when she rolls over in bed. She says it feels like the
room is spinning around her. Examination of her ears and cranial nerves is unremarkable. Given the likely diagnosis of benign
paroxysmal positional vertigo what is the most appropriate management?
Trial of prochlorperazine

Request MRI brain

Advise review by an optician

Perform Epley manoeuvre

Trial of cinnarizine

Next question

Benign paroxysmal positional vertigo


Benign paroxysmal positional vertigo (BPPV) is one of the most common causes of vertigo encountered. It is characterised by t he
sudden onset of dizziness and vertigo triggered by changes in head position. The average age of onset is 55 years and i t is less
common in younger patients.
Features

vertigo triggered by change in head position (e.g. rolling over in bed or gazing upwards)
may be associated with nausea
each episode typically lasts 10-20 seconds
positive Dix-Hallpike manoeuvre

BPPV has a good prognosis and usually resolves spontaneously after a few weeks to months. Symptomatic relief may be gained
by:

Epley manoeuvre (successful in around 80% of cases)


teaching the patient exercises they can do themselves at home, for example Brandt -Daroff exercises

Medication is often prescribed (e.g. Betahistine) but it tends to be of limited value.

Question 9 of 41
A 54-year-old woman with a history of hypertension presents to surgery. She has a 4 week history of hoarseness which followed
an upper respiratory tract infection 6 weeks ago. She is otherwise fit and well and is a non -smoker. What is the most appropriate
management?
Urgent chest x-ray

Check full blood count

Routine referral to ear, nose and throat

Reassure

Suggest chlorhexidine mouthwash

Next question
An urgent chest x-ray should be performed to direct fast-track referral

Hoarseness
Causes of hoarseness include:

voice overuse
smoking
viral illness
hypothyroidism
gastro-oesophageal reflux
laryngeal cancer
lung cancer

NICE guidelines on referral for suspect cancer suggest:

refer urgently for chest x-ray patients with hoarseness persisting for more than 3 weeks, particularly smokers aged older
than 50 years and heavy drinkers
if there is a positive finding on chest x-ray, refer urgently to a team specialising in the management of lung cancer
if the chest x-ray is normal, refer urgently to a team specialising in head and neck cancer

Question 10 of 41
A 21-year-old man presents with halitosis and mouth pain. Exam ination reveals very poor dental hygiene with bleeding gums and
widespread gingival ulceration. He has a temperature of 38.0C. You advise him to see a dentist. What other treatment options
should be offered?
Paracetamol + oral phenoxymethylpenicillin

Paracetamol + oral phenoxymethylpenicillin + chlorhexidine mouthwash

Paracetamol + chlorhexidine mouthwash

Paracetamol + oral metronidazole + chlorhexidine mouthwash

Paracetamol + oral metronidazole

Next question
This man has acute necrotizing ulcerative gingivitis with systemic upset. Treatment should be commenced whilst he is awaiting to
see a dentist.

Gingivitis
Gingivitis is usually secondary to poor dental hygiene. Clinical presentation may range from simple gingivitis (painless, red

swelling of the gum margin which bleeds on contact) to acute necrotizing ulcerative gingivitis (painful bleeding gums with halito sis
and punched-out ulcers on the gums).
If a patient presents with acute necrotizing ulcerative gingivitis CKS recommend the following management:

refer the patient to a dentist, meanwhile the following is recommended:


oral metronidazole* for 3 days
chlorhexidine (0.12% or 0.2%) or hydrogen peroxide 6% mouth wash
simple analgesia

*the BNF also suggest that amoxicillin may be used

Gingivitis and periodontitis - Summary


Gingivitis is inflammation of the gums due to any cause.
Acute necrotizing ulcerative gingivitis (ANUG) is a progressive, painful, acute bacterial infection
of the gums with ulceration, swelling, and sloughing off of dead tissue.
Periodontitis is extension of the gingival inflammation to involve the periodontal ligament and
bone.
The primary cause of plaque-associated gingivitis and periodontitis is ineffective oral hygiene
allowing dental plaque to accumulate. The most important risk factors for plaque-associated
gingivitis and periodontitis are:
Ineffective oral hygiene.
Cigarette smoking.
Diabetes mellitus.
Plaque-associated gingivitis may progress to periodontitis if bacterial plaque is not removed.
Periodontitis, if untreated, can progressively lead to complications including:
Damage to the periodontal attachment and alveolar bone.
Recurrent gum abscesses.
Detachment of the gum from the tooth with the formation of periodontal pockets.
Loss of multiple teeth.
Gingivitis is painless and discomfort from gums is rare. The main features are:

Reddening and swelling of the gum margins.


Bleeding of gums with tooth brushing, flossing, or gentle probing.
In periodontitis, symptoms (including pain) are commonly absent, but some people have:
Halitosis.
Foul taste in the mouth.
Recession and associated root sensitivity.
Drifting/loosening of teeth causing difficulty in eating.
Periodontal abscess, which may cause pain.
Acute necrotizing ulcerative gingivitis is characterized by acutely painful, bleeding gums;
excessive salivation; severe halitosis; difficulty in swallowing or talking; metallic taste; anorexia;
and malaise or fever, which may reach 39C. 'Punched-out' ulcers covered with a white
pseudomembrane may be present on the gums between the teeth but rarely on other parts of the
oral mucosa.
Acute necrotizing ulcerative gingivitis should be referred to a dentist for urgent assessment and
management.
Red flags that suggest serious conditions such as aggressive periodontitis and oral malignancy
include:
Rapid progression of periodontitis, or periodontitis in children or young adults.
Unexplained ulceration of the oral mucosa persisting for more than 3 weeks.
Oral swellings persisting for more than 3 weeks.
All unexplained red, or red and white patches (including suspected lichen planus) on the oral
mucosa that are painful or swollen or bleeding.
Unexplained tooth mobility persisting for more than 3 weeks.
The level of suspicion is further increased if the person is a heavy smoker and/or heavy alcohol
drinker.

Confirmation of the diagnosis of periodontitis requires dental examination with periodontal


probing and radiography. Oral swabs for culture and sensitivity provide no useful information and
are not recommended.
Management of plaque-associated gingivitis and periodontitis includes:
Professional scaling and polishing, root surface instrumentation, and sometimes surgical
procedures to remove plaque.
Good oral hygiene.
Smoking cessation advice, if appropriate.

Question 11 of 41
A 46-year-old woman presents with tinnitus (worse on the right side), episodic vertigo and hearing loss. Otoscopy is
unremarkable. An audiogram shows the following:

What does the audiogram show?


Bilateral mixed hearing loss

Right sensorineural hearing loss

Right conductive hearing loss

Bilateral conductive hearing loss

Bilateral sensorineural hearing loss

Next question

Audiograms
Audiograms are usually the first-line investigation that is performed when a patient complains of hearing difficulties. They are
relatively easy to interpret as long as some simple rules are followed:

anything above the 20dB line is essentially normal (marked in red on the blank audiogram below)
in sensorineural hearing loss both air and bone conduction are impaired
in conductive hearing loss only air conduction is impaired
in mixed hearing loss both air and bone conduction are impaired, with air conduction often being 'worse' than bone

Question 12 of 41
Which one of the following statements regarding Meniere's disease is correct?
More common in patients from the Indian Subcontinent

Symptoms resolve in the majority of patients after 6 -12 months

It is very rare that patients develop permanent hearing loss

More common in children

Approximately equal incidence in males and females

Next question

Meniere's disease
Meniere's disease is a disorder of the inner ear of unknown cause. It is characterised by excessive pressure and progressive
dilation of the endolymphatic system. It is more common in middle -aged adults but may be seen at any age. Meniere's disease
has a similar prevalence in both men and women.
Features

recurrent episodes of vertigo, tinnitus and hearing loss (sensorineural). Vertigo is usually the prominent symptom
a sensation of aural fullness or pressure is now recognised as being common
other features include nystagmus and a positive Romberg test
episodes last minutes to hours
typically symptoms are unilateral but bilateral symptoms may develop after a number of years

Natural history

symptoms resolve in the majority of patients after 5 -10 years


some patients may be left with hearing loss
psychological distress is common

Management

ENT assessment is required to confirm the diagnosis


patients should inform the DVLA. The current advice is to cease driving until satisfactory control of symptoms is achieved
acute attacks: buccal or intramuscular prochlorperazine. Admission is sometimes required
prevention: betahistine may be of benefit

Question 13 of 41
A 42-year-old woman presents as she has noticed a 'droop' in the right side of her face since she woke up this morning. There is
no associated limb weakness, dysphagia or visual disturbance. On examination you notice right -sided upper and lower facial
paralysis. Which one of the following features would be most consistent with a diagnosis of Bell's palsy?
Vesicular rash around the ear

Hyperacusis

Sensory loss over the distribution of the facial nerve

Pins and needles in the right arm

Rhinorrhoea

Next question

A vesicular rash around the ear would suggest a diagnosis of Ramsey Hunt syndrome. Hyperacusis is seen in around a third of
patients.

Bell's palsy
Bell's palsy may be defined as an acute, unilateral, idiopathic, facial nerve paralysis. The aetiology is unknown although th e role of
the herpes simplex virus has been investigated previously.
Features

lower motor neuron facial nerve palsy - forehead affected*


patients may also notice post-auricular pain (may precede paralysis), altered taste, dry eyes, hyperacusis

Management

in the past a variety of treatment options have been proposed including no treatment, prednisolone only and a
combination of aciclovir and prednisolone
following a National Institute for Health randomised controlled trial it is now recommended that prednisolone 25mg bd for
10 days should be prescribed for patients within 72 hours of onset of Bell's palsy. Adding in aciclovir gives no additional
benefit

eye care is important - prescription of artificial tears and eye lubricants should be considered

Prognosis

if untreated around 15% of patients have permanent moderate to severe weakness

*upper motor neuron lesion 'spares' upper face

Question 14 of 41
A 30-year-old man presents with sneezing, nasal blockage and a constant runny nose. Which one of the following does not have a
role in the management of allergic rhinitis?
Oral decongestants

Oral corticosteroids

Intranasal corticosteroids

Oral antihistamines

Intranasal antihistamines

Next question

Allergic rhinitis
Allergic rhinitis is an inflammatory disorder of the nose where the nose become sensitized to allergens such as house dust mi tes
and grass, tree and weed pollens. It may be classified as follows, although the clinical usefulness of such classifications remains
doubtful:

seasonal: symptoms occur around the same time every year. Seasonal rhinitis which occurs secondary to pollens is
known as hay fever

perennial: symptoms occur throughout the year


occupational: symptoms follow exposure to particular allergens within the work place

Features

sneezing
bilateral nasal obstruction
clear nasal discharge
post-nasal drip
nasal pruritus

Management of allergic rhi nitis

allergen avoidance
oral or intranasal antihistamines are first line
intranasal corticosteroids
course of oral corticosteroids are occasionally needed
there may be a role for short courses of topical nasal decongestants (e.g. oxymetazoline). They should not be used for
prolonged periods as increasing doses are required to achieve the same effect (tachyphylaxis) and rebound hypertrophy
of the nasal mucosa may occur upon withdrawal

Question 15 of 41
A 45-year-old man presents with dizziness and ri ght-sided hearing loss. Which one of the following tests would most likely indicate
an acoustic neuroma?
Jerky nystagmus

Left homonymous hemianopia

Tongue deviated to the left

Fasciculation of the tongue

Absent corneal reflex

Next question
Loss of corneal reflex - think acoustic neuroma

Acoustic neuroma
Acoustic neuromas (more correctly called vestibular schwannomas) account for approximately five percent of intracranial tumours
and 90 percent of cerebellopontine angle
Features can be predicted by the affected cranial nerves

cranial nerve VIII: hearing loss, vertigo, tinnitus


cranial nerve V: absent corneal reflex

cranial nerve VII: facial palsy

Bilateral acoustic neuromas are seen in neurofibromatosis type 2


MRI of the cerebellopontine angle is the investigation of choice

Question 16 of 41
Which one of the following features is least consistent with a diagnosis of otosclerosis?
Tinnitus

Positive family history

Normal tympanic membrane

Conductive deafness

Onset after the age of 50 years

Next question

Otosclerosis
Otosclerosis describes the replacement of normal bone by vascular spongy bone. It causes a progressive conductive deafness
due to fixation of the stapes at the oval window. Otosclerosis is autosomal dominant and typically affects young adults
Onset is usually at 20-40 years - features include:

conductive deafness
tinnitus
normal tympanic membrane*
positive family history

Management

hearing aid
stapedectomy

*10% of patients may have a 'flamingo tinge', caused by hyperaemia

Question 17 of 41
A 19-year-old woman presents with hearing problems for the past six months. She initially thought it was due to wax but her
hearing has not improved after ear syringing. You perform an examination of his auditory system including Rinne's and Weber's
test:
Rinne's test :

L eft ear: air conduction > bone conduction

Right ear: air c onduction > bone conduction


Weber's test:

L ateralises to the left s ide

What do these tests imply?


Left sensorineural deafness

Right conductive deafness

Normal hearing

Right sensorineural deafness

Left conductive deafness

Next question
In Weber's test if there is a sensorineural problem the sound is localised to the unaffected side (left) indicating a problem on the
right side.

Rinne's and Weber's test


Performing both Rinne's and Weber's test allows differentiation of conductive and sensorineural deafness.
Rinne's test

tuning fork is placed over the mastoid process until the sound is no longer heard, followed by repositioning just over
external acoustic meatus
air conduction (AC) is normally better than bone conduction (BC)
if BC > AC then conductive deafness

Weber's test

tuning fork is placed in the middle of the forehead equidistant from the patient's ears
the patient is then asked which side is loudest
in unilateral sensorineural deafness, sound is localised to the unaffected side
in unilateral conductive deafness, sound is localised to the affected side

Question 18 of 41
Which one of the following patients is most likely to have nasal polyps?
A 40-year-old man

A 40-year-old woman

An 8-year-old girl

An 80-year-old woman

An 8-year-old boy

Next question
Nasal polyps are most common in male adults

Nasal polyps
Around in 1% of adults in the UK have nasal polyps. They are around 2 -4 times more common in men and are not commonly seen
in children or the elderly.
Associations

asthma* (particularly late-onset asthma)


aspirin sensitivity*
infective sinusitis
cystic fibrosis
Kartagener's syndrome
Churg-Strauss syndrome

Features

nasal obstruction
rhinorrhoea, sneezing
poor sense of taste and smell

Unusual features which always require further investig ation include unilateral symptoms or bleeding.
Management

all patients with suspected nasal polyps should be referred to ENT for a full examination
topical corticosteroids shrink polyp size in around 80% of patients

*the association of asthma, aspirin sensitivity and nasal polyposis is known as Samter's triad

Question 19 of 41
A 61-year-old woman presents with bilateral tinnitus. She reports no change in her hearing or other ear-related symptoms. Ear
and cranial nerve examination is unremarkable. Which medication is she most likely to have recently started?
Ciprofloxacin

Nifedipine

Repaglinide

Quinine

Bendroflumethiazide

Next question

Tinnitus
Causes of tinnitus include:

Meniere's disease

Otosclerosis

Acoustic
neuroma
Hearing loss
Drugs

Associated w ith hearing loss, v ertigo, tinnitus and sensation of fullness or pressure in one or both
ears
Onset is usually at 20-40 years
Conductive deafness
Tinnitus
Normal tympanic membrane*
Positive family history
Hearing loss, vertigo, tinnitus
Absent corneal reflex is important sign
Associated with neurofibromatosis type 2
Causes include excessive loud noise and presbycusis
Aspirin
Aminoglycosides
Loop diuretics
Quinine

Other causes include

impacted ear wax


chronic suppurative otitis media

*10% of patients may have a 'flamingo tinge', caused by hyperaemia

Question 20 of 41
A 71-year-old man presents with two year history of intermittent problems with swallowing. His wife has also noticed he has
halitosis and is coughing at night. He has a past medical history of type 2 diabetes mellitus but states he is otherwise well. Of note
his weight is stable and he has a good appetite. Clinical examination is unremarkable. What is the most likely diagnosis?
Oesophageal cancer

Hiatus hernia

Pharyngeal pouch

Oesophageal candidiasis

Benign oesophageal stricture

Next question

Given the two year history and good health oesophageal cancer is much less likely

Pharyngeal pouch
A pharyngeal pouch is a posteromedial diverticulum through Killian's dehiscence. Killian's dehiscence is a triangular area in the
wall of the pharynx between the thyropharyngeus and cricopharyngeus muscles. It is more common in older patients and is 5
times more common in men
Features

dysphagia
regurgitation
aspiration
neck swelling which gurgles on palpation
halitosis

Image used on license from Radiopaedia

Still image taken from a barium swallow with fluoroscopy. During swallowing an outpouching of the posterior hypopharyngeal wall is visualised at the
lev el C5-C6, right above the upper oesophageal sphincter

Question 21 of 41
A 41-year-old woman presents with a sore throat. Examination of the throat reveals:

What is the most likely diagnosis?


Tonsillar carcinoma

Peritonsillar abscess (quinsy)

Acute tonsillitis

Infectious mononucleosis

Retropharyngeal abscess

Next question
Infectious mononucleosis is a possibility but a simple tonsillitis is the most likely diagnosis.

Tonsillitis and tonsillectomy


Complications of tonsillitis include:

otitis media
quinsy - peritonsillar abscess
rheumatic fever and glomerulonephritis very rarely

The indications for tonsillectomy are controversial. NICE recommend that surgery should be considered only if the person meet s
all of the following criteria

sore throats are due to tonsillitis (i.e. not recurrent upper respiratory tract infe ctions)
the person has five or more episodes of sore throat per year
symptoms have been occurring for at least a year
the episodes of sore throat are disabling and prevent normal functioning

Other established indications for a tonsillectomy include

recurrent febrile convulsions secondary to episodes of tonsillitis


obstructive sleep apnoea, stridor or dysphagia secondary to enlarged tonsils
peritonsillar abscess (quinsy) if unresponsive to standard treatment

Complications of tonsillectomy

primary (< 24 hours): haemorrhage in 2-3% (most commonly due to inadequate haemostasis), pain
secondary (24 hours to 10 days): haemorrhage (most commonly due to infection), pain

Question 22 of 41
A 24-year-old man who is suffering from sinusitis asks about using Sudafed (pseudoephedrine). Which one of the following
medications would make the use of Sudafed contraindicated?
Sodium valproate

Monoamine oxidase inhibitor

Salbutamol

Triptan

Selective serotonin reuptake inhibitor

Next question
A monoamine oxidase inhibitor combined with pseudoephedrine could potentially cause a hypertensive crisis.

Sinusitis
Sinusitis describes an inflammation of the mucous membranes of the paranasal sinuses. The sinuses are usually sterile - the most
common infectious agents seen in acute sinusitis are Streptococcus pneumoniae, Haemophilus influenzae and rhinoviruses.
Predisposing factors include:

nasal obstruction e.g. Septal deviation or nasal polyps


recent local infection e.g. Rhinitis or dental extraction
swimming/diving
smoking

Features

facial pain: typically frontal pressure pain which is worse on bending forward
nasal discharge: usually thick and purulent
nasal obstruction: e.g. 'mouth breathing'
post-nasal drip: may produce chronic cough

Management of acute sinusitis

analgesia
intranasal decongestants
oral antibiotics are not normally required but may be given for severe presentations. Amoxicillin is currently first -line

Management of recurrent or chronic sinusitis

treat any acute element as above


intranasal corticosteroids are often beneficial
referral to ENT may be appropriate

Sinusitis - Summary
Sinusitis is defined as inflammation of the mucosal lining of the paranasal sinuses.
Acute sinusitis is usually triggered by a viral upper respiratory tract infection and is defined by
symptoms that last for less than 12 weeks.
In adults it is diagnosed by the presence of nasal blockage or discoloured nasal discharge with
facial pain/pressure (or headache) and/or reduction of the sense of smell.
In children it is diagnosed by the presence of nasal blockage or discoloured nasal discharge with
facial pain/pressure (or headache) and/or cough.
Examination is of limited value but may reveal the presence of purulent discharge, swelling of the
nasal mucosa, tenderness over the sinuses, and fever.
Chronic sinusitis is defined by symptoms which last for longer than 12 weeks.
In adults it is diagnosed by the presence of nasal blockage or nasal discharge with facial pain (or
pressure) and/or reduction of the sense of smell.
In children it is diagnosed by the presence of nasal blockage or nasal discharge with facial pain
(or pressure) and/or cough.
Recurrent episodes of acute sinusitis may also occur, with worsening symptoms above the normal
background level of persistent symptoms.
Management of acute sinusitis includes:
Advice about the natural course and cause of the infection.

Recommending measures to relieve symptoms, such as analgesia for pain or fever, an intranasal
decongestant, irrigation of the nose with saline solution, application of warm face packs, drinking
adequate fluids, and rest.
Considering the need for antibiotics.
Considering high-dose intranasal corticosteroids in adults with more severe or prolonged
symptoms.
Management of chronic sinusitis includes:
Advice that chronic sinusitis may last several months, but does not usually require referral.
Recommending measures to relieve symptoms, such as analgesia for pain or fever, occasional use
of an intranasal decongestant, irrigation of the nose with saline solution, and application of warm
face packs.
Managing any associated disorder, such as allergic rhinitis, asthma, or dental infection.
Advising good dental hygiene and smoking cessation, if appropriate.
Considering the need for intranasal corticosteroids (especially if there is suspicion of an allergic
cause).
Considering the need for long-term antibiotics (and starting treatment only after discussing this
with a specialist).
Urgent hospital admission should be arranged if there is severe systemic infection, or if a
complication of acute sinusitis develops, such as orbital or intracranial involvement.
Urgent referral to an Ear, Nose, and Throat (ENT) specialist should be considered if there is a
suspected sinonasal tumour (persistent unilateral symptoms, such as bloodstained discharge,
crusting, non-tender facial pain, facial swelling, or unilateral nasal polyps).
Routine referral to ENT should be considered in chronic sinusitis if the person has:
Frequent, recurrent, troublesome episodes of acute sinusitis.
Unremitting or progressive facial pain.
Nasal polyps which are causing significant nasal obstruction.
Had a trial of intranasal corticosteroids for 3 months which was ineffective

Question 1 of 19
A 7-year-old girl is brought to surgery due to a sore throat. She has a temperature of 39.2C and is not eating due to the pain,
although she is tolerating fluids. She has had no other related symptoms such as a cough or a rash. Her heart rate is 120/min and
auscultation of the chest is unremarkable. The tonsils are covered in exudate bilaterally. Examination of the ears is unremarkable.
Other than supportive treatment, what is the most appropriate management?
Erythromycin for 10 days

Amoxicillin for 7 days

Antibiotics are not indicated

Phenoxymethylpenicillin for 10 days

Phenoxymethylpenicillin for 5 days

Next question

This girl has marked systemic upset and should be treated with antibiotics. A 7 or 10 day course of antibiotics is appropriat e to
ensure eradication of possible Streptococcus infection. Phenoxymethylpenicillin is the first-line antibiotic choice in the BNF

Sore throat
Sore throat encompasses pharyngitis, tonsillitis, laryngitis
Clinical Knowledge Summaries recommend:

throat swabs and rapid antigen tests should not be carried out routinely in patients with a sore throat

Management

paracetamol or ibuprofen for pain relief


antibiotics are not routinely indicated

NICE indications for antibiotics

features of marked systemic upset secondary to the acute sore throat


unilateral peritonsillitis
a history of rheumatic fever
an increased risk from acute infection (such as a child with diabetes mellitus or immunodeficiency)
patients with acute sore throat/acute pharyngitis/acute tonsillitis when 3 or more Centor criteria are p resent

The Centor criteria* are as follows:

presence of tonsillar exudate


tender anterior cervical lymphadenopathy or lymphadenitis
history of fever
absence of cough

If antibiotics are indicated then either phenoxymethylpenicillin or erythromycin (if the patient is penicillin allergic) should be given.
Either a 7 or 10 day course should be given

*if 3 or more of the criteria are present there is a 40 -60% chance the sore throat is caused by Group A beta -haemolytic
Streptococcus

Question 2 of 19
A 60-year-old man is diagnosed with Bell's palsy. What is the current evidenced base approach to the management of this
condition?
Refer for urgent surgical decompression

Aciclovir

No treatment

Aciclovir + prednisolone

Prednisolone

Next question
Eye care is also very important.

Bell's palsy
Bell's palsy may be defined as an acute, unilateral, idiopathic, facial nerve paralysis. The aetiology is unknown although th e role of
the herpes simplex virus has been investigated previously.
Features

lower motor neuron facial nerve palsy - forehead affected*


patients may also notice post-auricular pain (may precede paralysis), altered taste, dry eyes, hyperacusis

Management

in the past a variety of treatment options have been proposed including no treatment, prednisolone only and a
combination of aciclovir and prednisolone
following a National Institute for Health randomised controlled trial it is now recommended that prednisolone 25mg bd for
10 days should be prescribed for patients within 72 ho urs of onset of Bell's palsy. Adding in aciclovir gives no additional
benefit
eye care is important - prescription of artificial tears and eye lubricants should be considered

Prognosis

if untreated around 15% of patients have permanent moderate to severe weakness

*upper motor neuron lesion 'spares' upper face

Question 3 of 19
You see a 3-year-old boy as a follow-up appointment. Two weeks ago he presented with left-sided otalgia associated with a
purulent discharge. You prescribed amoxicillin and arranged to see him today. His mum reports that he is much better and says

she has managed to keep the ear dry. On examination of the left side a perforation of the tympanic membrane is noted. What is
the most appropriate action?
Advise to keep ear dry and see in a further 4 weeks time

Prescribe gentamicin ear drops to prevent infection + see in a further 6 weeks time

Advise to keep ear dry and see in a further 12 weeks time

Refer to ENT

Prescribe prophylactic dose amoxicillin to prevent infection + see in a further 4 weeks time

Next question

When he presented initially with the perforation this boy was given amoxicillin which is consistent with NICE guidelines. The re is
no indication for continuing the antibiotics if the ear is dry.
If there is still a perforation when the boy is reviewed in 4 weeks time (i.e. 6 weeks since the perforation occurred) then E NT
referral should be considered.

Perforated tympanic membrane


The most common cause of a perforated tympanic membrane is infection. Other causes include barotrauma or direct trauma.
A perforated tympanic membrane may lead to hearing loss depending on the size and also increase the risk of otitis media.
Management

no treatment is needed in the majority of cases as the tympanic membrane will usually heal after 6 -8 weeks. It is
advisable to avoid getting water in the ear during this time
it is common practice to prescribe antibiotics to perforations which occur following an episode of acute otitis media. NICE
support this approach in the 2008 Respiratory tract infection guidelines
myringoplasty may be performed if the tympanic membrane does not heal by itself

Question 1 of 16
A 44-year-old man asks for advice. He is due to go on a long bus journey but suffers from debilitating motion sickness. Which one
of the following medications is most likely to prevent motion sickness?
Cyclizine

Chlorpromazine

Metoclopramide

Prochlorperazine

Domperidone

Next question

Motion sickness - hyoscine > cyclizine > promethazine

Motion sickness
Motion sickness describes the nausea and vomiting which occurs when an apparent discrepancy exists between visually
perceived movement and the vestibular systems sense of movement
Management

the BNF recommends hyoscine (e.g. transdermal patch) as being the most effective treatment. Use is limited due to side effects
non-sedating antihistamines such as cyclizine or cinnarizine are recommended in preference to sedating prepa ration such
as promethazine

Question 2 of 16
A 25-year-old woman presents with recurrent attacks of 'dizziness'. These attacks typically last around 30 -60 minutes and occur
every few days or so. During an attack 'the room seems to be spinning' and the p atient often feels sick. These episodes are often
accompanied by a 'roaring' sensation in the left ear. Otoscopy is normal but Weber's test localises to the right ear. What is the
most likely diagnosis?
Acoustic neuroma

Vestibular neuritis

Benign paroxysmal positional vertigo

Multiple sclerosis

Meniere's disease

Next question
In sensorineural hearing loss Weber's test localises to the contralateral ear.

Meniere's disease
Meniere's disease is a disorder of the inner ear of unknown cause. It is characterised by excessive pressure and progressive
dilation of the endolymphatic system. It is more common in middle -aged adults but may be seen at any age. Meniere's disease
has a similar prevalence in both men and women.
Features

recurrent episodes of vertigo, tinnitus and hearing loss (sensorineural). Vertigo is usually the prominent symptom
a sensation of aural fullness or pressure is now recognised as being common
other features include nystagmus and a positive Romberg test
episodes last minutes to hours
typically symptoms are unilateral but bilateral symptoms may develop after a number of years

Natural history

symptoms resolve in the majority of patients after 5 -10 years


some patients may be left with hearing loss
psychological distress is common

Management

ENT assessment is required to confirm the diagnosis


patients should inform the DVLA. The current advice is to cease driving until satisfactory control of symptoms is achieved
acute attacks: buccal or intramuscular prochlorperazine. Admission is sometimes required
prevention: betahistine may be of benefit

Question 3 of 16
A 40-year-old musician complains of problems detecting pitch when he is playing the violin. You arrange an audiogram:

What does the audiogram show?


Right mixed hearing loss

Right sensorineural hearing loss

Right conductive hearing loss

Left conductive hearing loss

Bilateral mixed hearing loss

Next question

Audiograms
Audiograms are usually the first-line investigation that is performed when a patient complains of hearing difficulties. They are
relatively easy to interpret as long as some simple rules are followed:

anything above the 20dB line is essentially normal (marked in red on the blank audiogram below)
in sensorineural hearing loss both air and bone conduction are impaired
in conductive hearing loss only air conduction is impaired
in mixed hearing loss both air and bone conduction are impaired, with air conduction often being 'worse' than bone

Question 4 of 16

This 21-year-old woman has a history of recurrent epistaxis:

Image used on license from DermNet NZ

What is the most likely underlying diagnosis?


Idiopathic thrombocytopenic purpura

Peutz-Jeghers syndrome

Anorexia nervosa

Combined oral contraceptive pill use

Hereditary haemorrhagic telangiectasia

Next question

Hereditary haemorrhagic telangiectasia


Also known as Osler-Weber-Rendu syndrome, hereditary haemorrhagic telangiectasia (HHT) is an autosomal dominant condition
characterised by (as the name suggests) multiple telangiectasia over the skin and mucous membranes. Twenty percent of cases
occur spontaneously without prior family history.
There are 4 main diagnostic criteria. If the patient has 2 then they are said to have a possible diagnosis of HHT. If they me et 3 or
more of the criteria they are said to have a definite diagnosis of HHT:

epistaxis : spontaneous, recurrent nosebl eeds


telangiectases: multiple at characteristic sites (lips, oral cavity, fingers, nose)

visceral lesions: for example gastrointestinal telangiectasia (with or without bleeding), pulmonary arteriovenous
malformations (AVM), hepatic AVM, cerebral AVM, spina l AVM
family history: a first-degree relative with HHT

Image used on license from Radiopaedia

The chest x-ray shows multiple pulmonary nodules representing arteriovenous malformations, the largest in the right mid-zone. The CT scan shows
multiple hepatic arteriovenous malformations

Hereditary haemorrhagic telangiectasia


What is hereditary haemorrhagic telangiectasia?
Hereditary haemorrhagic telangiectasia (HHT) is also known as Osler-Rendu-Weber syndrome.
It is a rare inherited disorder that affects blood vessels throughout the body and is
characterised by a tendency for bleeding (haemorrhage, American spelling hemorrhage), in
particular recurrent epistaxis (nosebleeds), and skin telangiectasia (skin lesions resulting from
dilation of blood vessels).
The diagnostic criteria for HHT are:
1. Spontaneous recurrent nosebleeds
2. Multiple telangiectases on skin and mucous membranes
3. Inolvement of internal organs
4. An affected parent, sibling or child

What causes HHT?


The two major types of HHT are HHT1 and HHT2. They are caused by mutations in the
endoglin (ENG) and activin receptor-like kinase type 1 (ACVLR1) genes found on chromosome
9 and 12 respectively. Two other genes have also been identified. A defect in just one of these
genes causes an abnormality in the formation of blood vessels, which may easily rupture and
bleed. These abnormal blood vessels are known as telangiectases, or arteriovenous
malformations (AVM) if larger blood vessels are involved.

Who gets HHT?


HHT is a rare autosomal dominant condition, which means that only one abnormal gene needs
to be inherited from one affected parent to express the disease. HHT is often passed from
generation to generation as each child of a person with HHT has a 50% chance of getting the
disease. However, the signs and symptoms of HHT within a single family can vary
considerably. One family member may suffer from severe recurrent nosebleeds whilst another
with HHT may have minimal symptoms.

What are the signs and symptoms of HHT?


The most common sign of HHT is telangiectases in the nose and the most common symptom is
recurring nosebleeds. The first sign of HHT usually does not occur until puberty or adulthood
with the average age of the first nosebleed occurring at 12 years. Bleeding may occur as often
as everyday or as infrequently as once a month. Recurrent nosebleeds are seen in 50 -80% of
patients with HHT.
Telangiectases in other parts of the body is not usually seen until after puberty and is most
apparent in people aged between 20 and 40 years. It occurs in about 95% of patients with
HHT. Telangiectasia of the skin and mucous membranes has the following characteristics.

Appearance of small red to purplish spots or dark red lacy lines on the skin and
mucous membranes

Lesions may occur anywhere but especially on the upper half of the body including the
face, inside the mouth and nostrils, lips, ears, conjunctiva of the eyes, forearms,
hands and fingers. They are often conspicuous in the nail beds.

Lesions may initially appear subtle but become quite prominent by late adulthood

Telangiectases on the skin and mouth can bleed but are less likely to than those in the
nose

In addition to visible telangiectases, abnormal blood vessel formation may occur in many other
organs. Telangiectases can be found anywhere in the gastrointestinal (GI) system, including
the oesophagus, stomach, and small and large intestines. GI bleeding occurs in about 25% of
patients with HHT and the risk is increased in patients older than 50 years. Black or bloody
stools and/or anaemia (low blood count) are the presenting symptoms. Other organs that may
be affected include the lungs (AVM in the lungs occur in about 30% of patients with HHT) and
central nervous system (brain and spinal AVM).

Hereditary haemorrhagic telangiectasia

Can HHT be treated?


HHT cannot be prevented but most cases can be treated symptomatically. One third of the
cases of HHT are mild, one third are moderate, and one third are severe. Mild cases usually
require no treatment. HHT should be treated if it is causing significant pro blems, such as
severe and/or frequent nosebleeds, or if there is a high risk of causing other problems, such
as a stroke from a lung AVM.

Nosebleeds can be treated with laser coagulation therapy or surgically with nasal
septum skin transplants (septal dermoplasty)

Telangiectases or lesions of the skin can be treated with cautery or dye laser surgery,
best performed by a dermatologist

GI bleeding causing anaemia is treated with iron replacement therapy. If this is


ineffective then blood transfusion and endoscopic treatments may be performed.

Question 5 of 16
Which one of the following viruses is associated with nasopharyngeal carcinoma?
Adenovirus

Rhinovirus

Herpes simplex virus

Epstein-Barr virus

Picornavirus

Next question
EBV: associated malignancies:

Burkitt's lymphoma
Hodgkin's lymphoma
nasopharyngeal carcinoma

Epstein-Barr virus: associated conditions


Malignancies associated with EBV infection

Burkitt's lymphoma*
Hodgkin's lymphoma
nasopharyngeal carcinoma
HIV-associated central nervous system lymphomas

The non-malignant condition hairy leukoplakia is also associated with EBV infection.
*EBV is currently thought to be associated with both African and sporadic Burkitt's

Question 6 of 16
Which one of the following statements regarding trigeminal neuralgia is correct?

Duloxetine is the first-line treatment

All patients with suspected trigeminal neuralgia should be referred to secondary care

The pain is commonly triggered by touching the skin

The pain is usually constant

It is bilateral in 30% of cases

Next question

The pain is often triggered by light touch, shaving, eating etc. Only around 10% of cases are bilateral.

Trigeminal neuralgia
Trigeminal neuralgia is a pain syndrome characterised by severe unilateral pain. The vast majority of cases are idiopathic but
compression of the trigeminal roots by tumours or vascular problems may occur
The International Headache Society defines trigeminal neuralgia as:

a unilateral disorder characterised by brief electric shock-like pains, abrupt in onset and termination, limited to one or
more divisions of the trigeminal nerve
the pain is commonly evoked by light touch, including washing, shaving, smoking, talking, and brushing the teeth (trigger
factors), and frequently occurs spontaneously
small areas in the nasolabial fold or chin may be particularly susceptible to the precipitation of pain (trigger areas)
the pains usually remit for variable periods

Management

carbamazepine is first-line
failure to respond to treatment or atypical features (e.g. < 50 years old) should prompt referral to neurology

Trigeminal neuralgia - Summary


Trigeminal neuralgia occurs in the distribution of one or more branches of the fifth (trigeminal)
cranial nerve.
Pain may occur infrequently (e.g. periods of remission may last years) or with a frequency of up
to hundreds of times a day.
In 8090% of cases, trigeminal neuralgia is thought to be caused by compression of the
trigeminal nerve by a loop of artery or vein.
Almost twice as many women are affected by trigeminal neuralgia than men. The incidence
increases with age and is rare in people younger than 40 years of age.
Complications include depression, and inability to eat causing weight loss.

Features of trigeminal neuralgia include paroxysmal attacks of pain which may be precipitated
from trigger areas or by trigger factors.
There is no clinically evident neurological deficit.
Attacks are stereotyped in the individual patient.
'Atypical' or 'mixed' trigeminal neuralgia occurs when there is a persistent discomfort between
paroxysms or sensory loss.
The drug of choice for the treatment of trigeminal neuralgia is carbamazepine:
Dose should be titrated until pain is relieved. In the majority of people a dose of 200 mg three or
four times a day is sufficient to prevent paroxysms of pain (maximum dosage 1600 mg daily).
If carbamazepine is inappropriate, ineffective, or not tolerated, specialist advice should be sought,
or referral made to a neurologist or a specialist in pain management. A daily pain diary may be
useful to help people learn to manage their pain.
When the pain is in remission, the dose of carbamazepine should be reduced, and gradually
withdrawn if the person remains pain-free for 1 month.
Referral to a neurosurgeon, neurologist, or specialist in pain management (with an interest in
trigeminal neuralgia) should be made if any of the following apply:
They have severe pain.
Their pain significantly limits their daily activities and participation.
Atypical clinical features (e.g. burning pain between paroxysms, loss of sensation, abnormal
neurological signs) are present.
Carbamazepine is inappropriate, ineffective, or not tolerated.
Trigeminal neuralgia occurs in a person younger than 40 years of age.

Question 7 of 16
Which one of the following medications is most useful for helping to prevent attacks of Meniere's disease?
Promethazine

Prochlorperazine

Betahistine

Chlorphenamine

Cinnarizine

Next question

Meniere's disease
Meniere's disease is a disorder of the inner ear of unknown cause. It is characterised by excessive pressure and progressive
dilation of the endolymphatic system. It is more common in middle -aged adults but may be seen at any age. Meniere's disease
has a similar prevalence in both men and women.
Features

recurrent episodes of vertigo, tinnitus and hearing loss (sensorineural). Vertigo is usually the prominent symptom
a sensation of aural fullness or pressure is now recognised as being common
other features include nystagmus and a positive Romberg test
episodes last minutes to hours
typically symptoms are unilateral but bilateral symptoms may develop after a number of years

Natural history

symptoms resolve in the majority of patients after 5 -10 years


some patients may be left with hearing loss
psychological distress is common

Management

ENT assessment is required to confirm the diagnosis


patients should inform the DVLA. The current advice is to cease driving until satisfactory control of symptoms is achieved
acute attacks: buccal or intramuscular prochlorperazine. Admission is sometimes required
prevention: betahistine may be of benefit

Question 8 of 16
Which one of the following conditions is least associated with nasal polyps?
Wegener's granulomatosis

Kartagener's syndrome

Asthma

Infective sinusitis

Cystic fibrosis

Next question

Nasal polyps
Around in 1% of adults in the UK have nasal polyps. They are around 2 -4 times more common in men and are not commonly seen
in children or the elderly.
Associations

asthma* (particularly late-onset asthma)


aspirin sensitivity*
infective sinusitis
cystic fibrosis
Kartagener's syndrome
Churg-Strauss syndrome

Features

nasal obstruction
rhinorrhoea, sneezing
poor sense of taste and smell

Unusual features which always require further investigation include unilateral symptoms or bleeding.
Management

all patients with suspected nasal polyps should be referred to ENT for a full examination
topical corticosteroids shrink polyp size in around 80% of patients

*the association of asthma, aspirin sensitivity and nasal polyposis is known as Samter's triad

Question 9 of 16
A 59-year-old man presents with recurrent attacks of vertigo and dizziness. These attacks are often precipitated by a change in
head position and typically last around half a minute. Examination of the cranial nerves and ears is unremarkable. His blood
pressure is 120/78 mmHg sitting and 116/76 mmHg standing. Given the likely diagnosis of benign paroxysmal positional vertigo,
what is the most appropriate next step to help confirm the diagnosis?
Epley manoeuvre

Tilt table test

Tympanometry

MRI of the cerebellopontine angle

Dix-Hallpike manoeuvre

Next question
This patient has classical symptoms of benign paroxysmal positional vertigo. A positive Dix-Hallpike manoeuvre is an appropriate
next step and would help support the diagnosis.
The change in blood pressure on standing is not significant.

Benign paroxysmal positional vertigo


Benign paroxysmal positional vertigo (BPPV) is one of the most common causes of vertigo encountered. It is characterised by the
sudden onset of dizziness and vertigo triggered by changes in head position. The average age of onset is 55 years and it is l ess
common in younger patients.
Features

vertigo triggered by change in head position (e.g. rolling over in bed or gazing upwards)
may be associated with nausea
each episode typically lasts 10-20 seconds
positive Dix-Hallpike manoeuvre

BPPV has a good prognosis and usually resolves spontaneously after a few weeks to months. Symptomatic relief may be gained
by:

Epley manoeuvre (successful in around 80% of cases)


teaching the patient exercises they can do themselves at home, for example Brandt -Daroff exercises

Medication is often prescribed (e.g. Betahistine) but it tends to be of limited value.

Question 10 of 16
You review a 25-year-old man who has allergic rhinits. He has been using intranasal oxymetazoline which he bought from the
local chemist for the past 10 days. What is the main side -effect of using topical decongestants for prolonged periods?
Permanent loss of smell

Infective sinusitis

Post-nasal drip

Tachyphylaxis

Necrosis of the nasal septum

Next question
After using topical decongestants for prolonged periods increasing doses are needed to provide the same effect, a phenomenon
known as tachyphylaxis.

Allergic rhinitis

Allergic rhinitis is an inflammatory disorder of the nose where the nose become sensitiz ed to allergens such as house dust mites
and grass, tree and weed pollens. It may be classified as follows, although the clinical usefulness of such classifications r emains
doubtful:

seasonal: symptoms occur around the same time every year. Seasonal rhinitis which occurs secondary to pollens is
known as hay fever
perennial: symptoms occur throughout the year
occupational: symptoms follow exposure to particular allergens within the work place

Features

sneezing
bilateral nasal obstruction
clear nasal discharge
post-nasal drip
nasal pruritus

Management of allergic rhinitis

allergen avoidance
oral or intranasal antihistamines are first line
intranasal corticosteroids
course of oral corticosteroids are occasionally needed
there may be a role for short courses of topical nasal decongestants (e.g. oxymetazoline). They should not be used for
prolonged periods as increasing doses are required to achieve the same effect (tachyphylaxis) and rebound hypertrophy
of the nasal mucosa may occur upon withdrawal

Question 11 of 16
A 23-year-old man presents with a 4 day history of an itchy and sore right ear. He has recently returned from holiday in Spain. On
examination the right ear canal is inflamed but no debris is seen. The tympanic membrane is clearly visible an d is unremarkable.
What is the most appropriate management?
Topical corticosteroid + aminoglycoside

Topical corticosteroid

Refer to ENT

Topical corticosteroid + clotrimazole

Oral flucloxacillin

Next question

This patient has otitis externa, which commonly develops after swimming on holiday. The first line management is either a topical
antibiotic or a combined topical antibiotic and steroid.

Otitis externa
Otitis externa is a common reason for primary care attendance in the UK.

Causes of otitis externa include:

infection: bacterial (Staphylococcus aureus, Pseudomonas aeruginosa) or fungal


seborrhoeic dermatitis
contact dermatitis (allergic and irritant)

Features

ear pain, itch, discharge


otoscopy: red, swollen, or eczematous canal

The recommend initial management of otitis externa is:

topical antibiotic or a combined topical antibiotic with steroid


if the tympanic membrane is perforated aminoglycosides are traditionally not used*
if there is canal debris then consider removal
if the canal is extensively swollen then an ear wick is sometimes inserted

Second line options include

consider contact dermatitis secondary to neomycin


oral antibiotics if the infection is spreading
taking a swab inside the ear canal
empirical use of an antifungal agent

Malignant otitis externa is more common in elderly diabetics. In this condition there is extension of infection into the bony ear canal
and the soft tissues deep to the bony canal. Intravenous antibiotics may be required.
*many ENT doctors disagree with this and feel that concerns about ototoxicity are unfounded

Question 12 of 16
A 26-year-old man presents to surgery for review. He was seen two weeks ago with a sore throat and advised to take paracetamol
and fluids. He says his sore throat has got worse. On examination temperature is 37.7C and pulse is 84 bpm. Visualisation of the
throat is as follows:

What is the most appropriate management?


Refer to ENT

Send infectious mononucleosis screen

Paracetamol + Difflam spray + review in 1 week

Oral penicillin V + review in 1 week

Oral amoxicillin + review in 1 week

Next question

A peritonsillar abscess (quinsy) can be seen on the right side - this needs to incised and drained

Tonsillitis and tonsillectomy


Complications of tonsillitis include:

otitis media
quinsy - peritonsillar abscess
rheumatic fever and glomerulonephritis very rarely

The indications for tonsillectomy are controversial. NICE recommend that surgery should be considered only if the person meets
all of the following criteria

sore throats are due to tonsillitis (i.e. not recurrent upper respiratory tract infections)
the person has five or more episodes of sore throat per year
symptoms have been occurring for at least a year
the episodes of sore throat are disabling and prevent normal functioning

Other established indications for a tonsillectomy include

recurrent febrile convulsions secondary to episodes of tonsillitis


obstructive sleep apnoea, stridor or dysphagia secondary to enlarged tonsil s
peritonsillar abscess (quinsy) if unresponsive to standard treatment

Complications of tonsillectomy

primary (< 24 hours): haemorrhage in 2-3% (most commonly due to inadequate haemostasis), pain
secondary (24 hours to 10 days): haemorrhage (most commonly due to infection), pain

Question 13 of 16
During a routine cranial nerve examination the following findings are observed:
Rinne's test:

A ir c onduction > bone conduction in both ears

Weber's test:

L oc alises to the right s ide

What do these tests imply?


Left conductive deafness

Normal hearing

Right conductive deafness

Right sensorineural deafness

Left sensorineural deafness

Next question

In Weber's test if there is a sensorineural problem the sound is localised to the unaffected side (right) indicating a problem on the
left side

Rinne's and Weber's test


Performing both Rinne's and Weber's test allows differentiation of conductive and sensorineural deafness.
Rinne's test

tuning fork is placed over the mastoid process until the sound is no longer heard, followed by repositioning just over
external acoustic meatus
air conduction (AC) is normally better than bone conduction (BC)
if BC > AC then conductive deafness

Weber's test

tuning fork is placed in the middle of the forehead equidistant from the patient's ears
the patient is then asked which side is loudest
in unilateral sensorineural deafness, sound is localised to the unaffected side
in unilateral conductive deafness, sound is localised to the affected side

Question 14 of 16
A 34-year-old man complains of a sore throat. Which one of the following is not part of the Centor criteria used to assess the
likelihood of a bacterial cause?
Fever

Tender anterior cervical lymphadenopathy

Duration > 5 days

Absence of cough

Presence of tonsillar exudate

Next question

If 3 or more of the 4 Centor criteria are present there is a 40 -60% chance the sore throat is caused by Group A beta -haemolytic
Streptococcus

Respiratory tract infections: NICE guidelines


NICE issued guidance in 2008 on the management of respiratory tract infection, focusing on the prescribing of antibiotics for selflimiting respiratory tract infections in adults and children in primary care
A no antibiotic prescribing or delayed antibiotic prescribing approach is generally recommended for patients with acute otiti s
media, acute sore throat/acute pharyngitis/acute tonsillitis, common cold, acute rhinosinusitis or acute cough/acute bronchitis.
However, an immediate antibiotic prescribing approach may be considered for:

children younger than 2 years with bilateral acute otitis media


children with otorrhoea who have acute otitis media
patients with acute sore throat/acute pharyngitis/acute tonsillitis when 3 or more Centor criteria are present

The Centor criteria* are as follows:

presence of tonsillar exudate


tender anterior cervical lymphadenopathy or lymphadenitis
history of fever
absence of cough

If the patient is deemed at risk of developing complications, an immediate antibiotic prescribing policy is recommended

are systemically very unwell


have symptoms and signs suggestive of serious illness and/or complications (particularly pneumonia, mastoiditis,
peritonsillar abscess, peritonsillar cel lulitis, intraorbital or intracranial complications)
are at high risk of serious complications because of pre -existing comorbidity. This includes patients with significant heart,
lung, renal, liver or neuromuscular disease, immunosuppression, cystic fibrosis, and young children who were born
prematurely
are older than 65 years with acute cough and two or more of the following, or older than 80 years with acute cough and
one or more of the following:
- hospitalisation in previous year
- type 1 or type 2 diabetes
- history of congestive heart failure
- current use of oral glucocorticoids

The guidelines also suggest that patients should be advised how long respiratory tract infections may last:

acute otitis media: 4 days


acute sore throat/acute pharyngitis/acute tonsillitis: 1 week
common cold: 1 1/2 weeks
acute rhinosinusitis: 2 1/2 weeks
acute cough/acute bronchitis: 3 weeks

*if 3 or more of the criteria are present there is a 40 -60% chance the sore throat is caused by Group A beta -haemolytic
Streptococcus

Question 15 of 16
A 30-year-old man presents with facial pain and a 'heavy head' sensation after having a cold. A diagnosis of acute sinusitis is
suspected. Which one of the following should be considered for symptomatic relief?
Intranasal decongestants

Intranasal corticosteroids

Oral antihistamine

Oral mucolytics

Steam inhalation

Next question

Analgesia is also important. Please see the CKS guidelines for more information.

Sinusitis
Sinusitis describes an inflammation of the mucous membranes of the paranasal sinuses. The sinuses are usually sterile - the most
common infectious agents seen in acute sinusitis are Streptococcus pneumoniae, Haemophilus influenzae and rhinoviruses.
Predisposing factors include:

nasal obstruction e.g. Septal deviation or nasal polyps


recent local infection e.g. Rhinitis or dental extraction
swimming/diving
smoking

Features

facial pain: typically frontal pressure pain which is worse on bending forward
nasal discharge: usually thick and purulent
nasal obstruction: e.g. 'mouth breathing'
post-nasal drip: may produce chronic cough

Management of acute sinusitis

analgesia
intranasal decongestants
oral antibiotics are not normally required but may be given for se vere presentations. Amoxicillin is currently first-line

Management of recurrent or chronic sinusitis

treat any acute element as above


intranasal corticosteroids are often beneficial
referral to ENT may be appropriate

Question 16 of 16
A 71-year-old man presents to surgery with his wife. She describes his hearing as having been 'terrible' for many years but
unfortunately it has recently got worse. Otoscopy shows bilateral mild otitis externa with wax blocking the view of the tympa nic
membranes. Treatment for otitis externa is given, following which you arrange an audiogram:

What does the audiogram show?


Left conductive hearing loss

Bilateral conductive hearing loss

Bilateral sensory hearing loss

Left mixed hearing loss

Left sensorineural hearing loss

Next question

Audiograms
Audiograms are usually the first-line investigation that is performed when a patient complains of hearing difficulties. They are
relatively easy to interpret as long as some simple rules are followed:

anything above the 20dB line is essentially normal (marked in red on the blank audiogram below)
in sensorineural hearing loss both air and bone conduction are impaired
in conductive hearing loss only air conduction is impaired
in mixed hearing loss both air and bone conduction are impaired, with air conduction often being 'worse' than bone

You might also like